LSAT and Law School Admissions Forum

Get expert LSAT preparation and law school admissions advice from PowerScore Test Preparation.

User avatar
 Dave Killoran
PowerScore Staff
  • PowerScore Staff
  • Posts: 5852
  • Joined: Mar 25, 2011
|
#87786
Complete Question Explanation
(The complete setup for this game can be found here: lsat/viewtopic.php?f=157&t=1677)

The correct answer choice is (D)

From the discussion of the grants-to-areas Numerical Distributions, only two distributions are possible: 2-1-1-1 and 2-2-1-1. Thus, answer choice (D), which would force a 2-2-2-1 distribution (2W, 2Y, 2M and 1T), cannot occur and is the correct answer.

Get the most out of your LSAT Prep Plus subscription.

Analyze and track your performance with our Testing and Analytics Package.